Όρια με ολοκληρώματα

Συντονιστές: grigkost, Κοτρώνης Αναστάσιος

Άβαταρ μέλους
Κοτρώνης Αναστάσιος
Επιμελητής
Δημοσιεύσεις: 3203
Εγγραφή: Κυρ Φεβ 22, 2009 11:11 pm
Τοποθεσία: Μπροστά στο πισί...
Επικοινωνία:

Όρια με ολοκληρώματα

#1

Μη αναγνωσμένη δημοσίευση από Κοτρώνης Αναστάσιος » Δευ Μάιος 03, 2010 10:41 pm

Μαζεύω εδώ κάποια όρια με ολοκληρώματα που είχα βάλει στα ποστ "Δυο όρια ολοκληρωμάτων" και "Άλλα τρια όρια ολοκληρωμάτων", τα οποία και σβήνω από εκεί.

Κάποια μπορoύν να λυθούν με τρόπους που σουλουπώνονται και στα λυκειακά πλαίσια και κάποια άλλα όχι.

Θα συμπληρώνω την εδώ λίστα για να είναι όλα μαζεμένα, αφού το θέματα είναι σχετικά.


\displaystyle{1)} \displaystyle{\lim_{n\to\infty} {\int_0}^1\frac{1+n x^2 }{(1+x^2)^{n}}dx} \displaystyle{\boxed{\checkmark}}


\displaystyle{2)} \displaystyle{\lim_{n \to \infty} {\int_0}^\infty  \frac{n \sin\big(\frac{x}{n}\big)}{x(1+x^2)}  dx} \displaystyle{\boxed{\checkmark}}

\displaystyle{3)} \displaystyle{\lim_{t\to+\infty}\int_{1}^{2}\ln\Big(x+\frac{x^{5}}{t}\Big)\,dx}, (βγαίνει και λυκειακά) \displaystyle{\boxed{\checkmark}}

\displaystyle{4)} \displaystyle{\lim_{t\to+\infty} 
\int_{0}^{\pi/2}e^{-t\sin x}\,dx}, (βγαίνει και λυκειακά) \displaystyle{\boxed{\checkmark}}

\displaystyle{5)} \displaystyle{\lim_{n\to+\infty}\Bigg(\frac{1}{3\pi}\int_{\pi}^{2\pi}\frac{x}{\tan^{-1}(nx)}\,dx\Bigg)^{n}}.\displaystyle{\boxed{\checkmark}}

\displaystyle{6)} Αν η f:[0,1]\to\mathbb{R} είναι συνεχής και a,b>0, να βρεθεί το όριο \displaystyle{\lim_{t\to0^{+}}\int_{at}^{bt}\frac{f(x)}{x}\,dx}. (βγαίνει και λυκειακά) \displaystyle{\boxed{\checkmark}}

\displaystyle{7)} \displaystyle{\lim_{n\to+\infty}\int_{0}^{1}nx(1-x^{2})^{n}\,dx}. \displaystyle{\boxed{\checkmark}}

\displaystyle{8)} \displaystyle{\lim_{n\to+\infty}\int_{a}^{b}\frac{\sin nx}{x}\,dx} όπου 0<a<b. \displaystyle{\boxed{\checkmark}}


\displaystyle{9)} \bullet Έστω f συνεχής στο [0,1] και \mathrm{P(x),Q(x)} μη σταθερά πολυώνυμα με \mathrm{deg P(x)}=\mathrm{deg Q(x)} και με το \mathrm{Q} να έχει θετικό μεγιστοβάθμιο συντελεστή.

Ας υπολογισθεί το όριο \begin{center}\displaystyle{\lim_{n\to+\infty}\int_{0}^{1}\mathrm{P(n)}x^{\mathrm{Q(n)}}f(x)\,dx}\end{canter}.
(Αυτή είναι δικό μου μαγείρεμα και γενικεύει το παρακάτω. Ελπίζω να μην έχει γίνει καμμία πατάτα)

\bullet Ας βρεθεί τώρα το όριο \displaystyle{\lim_{n\to+\infty}\frac{\displaystyle\int_{0}^{1}x^{n}f(x)\,dx}{\displaystyle\int_{0}^{1}x^{n}\,dx}}. (Αυτό είναι από το Νεγρεπόντη) \displaystyle{\boxed{\checkmark}}

\displaystyle{10)} Με τις ίδιες προϋποθέσεις όπως και στο 9), ας υπολογισθεί το όριο

\displaystyle{\lim_{n\to+\infty}\int_{0}^{1}\mathrm{P(n)}e^{-\mathrm{Q(n)}x}f(x)\,dx}.
(Και αυτή είναι δικό μου μαγείρεμα. Ελπίζω πάλι...).

\displaystyle{11)} \displaystyle{\lim_{n\to+\infty}\Big(\frac{1}{\pi}\int_{0}^{\pi}\frac{\sqrt{n}}{\sqrt{n}+\cos x}\,dx\Big)^{n}}

\displaystyle{12)} Τούτο εδώ είναι απολύτως σχετικό με τα προβλήματα που τίθενται
\displaystyle{\color{blue}\bullet} εδώ
\displaystyle{\color{blue}\bullet} εδώ και
\displaystyle{\color{blue}\bullet} εδώ.

Έστω f:[0,1]\to(0,+\infty) συνεχής. Δείξτε ότι για κάθε n\in\mathbb{N} υπάρχει \theta_{n} με

\displaystyle{\frac{1}{n}\int_{0}^{1}f(x)\,dx=\int_{0}^{\theta_{n}}f(x)\,dx+\int_{1-\theta_{n}}^{1}f(x)\,dx} και υπολογίστε το όριο \displaystyle{\lim_{n\to+\infty}n\theta_{n}}.
τελευταία επεξεργασία από Κοτρώνης Αναστάσιος σε Σάβ Νοέμ 06, 2010 4:19 pm, έχει επεξεργασθεί 20 φορές συνολικά.


Εσύ....; Θα γίνεις κανίβαλος....;
Άβαταρ μέλους
chris_gatos
Επιμελητής
Δημοσιεύσεις: 6962
Εγγραφή: Κυρ Δεκ 21, 2008 9:03 pm
Τοποθεσία: Ανθούπολη

Re: Όρια με ολοκληρώματα

#2

Μη αναγνωσμένη δημοσίευση από chris_gatos » Δευ Μάιος 03, 2010 10:52 pm

Θα ξεκινήσω με το 7)

Θέτω x=sint
Αν x=0 => t=0
Aν x=1 => t=π/2
dx=costdt

Αρα:

\displaystyle{ 
\int\limits_0^1 {nx(1 - x^2 )^n dx = n\int\limits_0^{\frac{\pi }{2}} {\sin t} (1 - \sin ^2 t)^n \cos tdt =  - n\int\limits_0^{\frac{\pi }{2}} {\cos ^{2n + 1} td(\cos t) =  - n\left[ {\frac{{\cos ^{2n + 2} t}}{{2n + 2}}} \right]_0^{\frac{\pi }{2}}  = \frac{n}{{2n + 2}}} }  
}

Λαμβάνοντας όριο για n->+00 έχω όριο το 1/2.


Χρήστος Κυριαζής
Άβαταρ μέλους
mathxl
Δημοσιεύσεις: 6736
Εγγραφή: Τρί Δεκ 23, 2008 3:49 pm
Τοποθεσία: Σιδηρόκαστρο
Επικοινωνία:

Re: Όρια με ολοκληρώματα

#3

Μη αναγνωσμένη δημοσίευση από mathxl » Τρί Μάιος 04, 2010 1:04 am

Ας συνεχίσυμε την αντίστροφη μέτρηση

6) Απώλεια τύπου συνέχεια σε κλειστό = θεώρημα μέγιστης ελάχιστης τιμής
Έστω α < β (χβγ) η f(x)/x είναι συνεχής στο \displaystyle{0 < at \le x \le bt}
ως πηλίκο συνεχών σε αυτό, άρα έχει ελάχιστη τιμή m και μέγιστη τιμή Μ
Άρα
\displaystyle{m \le \frac{{f\left( x \right)}}{x} \le M \Rightarrow m\left( {b - a} \right)t \le \int\limits_{at}^{bt} {\frac{{f(x)}}{x}} \,dx \le M\left( {b - a} \right)t}
με ΚΠ το όριο είναι 0


Ποτε δεν κάνω λάθος! Μια φορά νομιζα πως είχα κάνει, αλλά τελικά έκανα λάθος!
Απ' τα τσακάλια δεν γλυτώνεις μ' ευχές η παρακάλια. Κ. Βάρναλης
Aπέναντι στις αξίες σου να είσαι ανυποχώρητος

Ενεργό μέλος από 23-12-2008 ως και 17-8-2014 (δεν θα απαντήσω σε πμ)
Άβαταρ μέλους
Κοτρώνης Αναστάσιος
Επιμελητής
Δημοσιεύσεις: 3203
Εγγραφή: Κυρ Φεβ 22, 2009 11:11 pm
Τοποθεσία: Μπροστά στο πισί...
Επικοινωνία:

Re: Όρια με ολοκληρώματα

#4

Μη αναγνωσμένη δημοσίευση από Κοτρώνης Αναστάσιος » Τρί Μάιος 04, 2010 1:35 am

mathxl έγραψε:Ας συνεχίσυμε την αντίστροφη μέτρηση

6) Απώλεια τύπου συνέχεια σε κλειστό = θεώρημα μέγιστης ελάχιστης τιμής
Έστω α < β (χβγ) η f(x)/x είναι συνεχής στο \displaystyle{0 < at \le x \le bt}
ως πηλίκο συνεχών σε αυτό, άρα έχει ελάχιστη τιμή m και μέγιστη τιμή Μ
Άρα
\displaystyle{m \le \frac{{f\left( x \right)}}{x} \le M \Rightarrow m\left( {b - a} \right)t \le \int\limits_{at}^{bt} {\frac{{f(x)}}{x}} \,dx \le M\left( {b - a} \right)t}
με ΚΠ το όριο είναι 0
Μου πήρε λίγη ώρα να βρώ το λάθος... :D. Το λεπτό σημείο εδώ είναι ότι M=M(t) και m=m(t) οπότε το κριτήριο παρεμβολής δε δουλεύει.


Εσύ....; Θα γίνεις κανίβαλος....;
Άβαταρ μέλους
mathxl
Δημοσιεύσεις: 6736
Εγγραφή: Τρί Δεκ 23, 2008 3:49 pm
Τοποθεσία: Σιδηρόκαστρο
Επικοινωνία:

Re: Όρια με ολοκληρώματα

#5

Μη αναγνωσμένη δημοσίευση από mathxl » Τρί Μάιος 04, 2010 1:45 am

Οκ Τάσο θα το δω κα θα επανορθώσω αύριο αν προλάβω. Καληνύχτα !


Ποτε δεν κάνω λάθος! Μια φορά νομιζα πως είχα κάνει, αλλά τελικά έκανα λάθος!
Απ' τα τσακάλια δεν γλυτώνεις μ' ευχές η παρακάλια. Κ. Βάρναλης
Aπέναντι στις αξίες σου να είσαι ανυποχώρητος

Ενεργό μέλος από 23-12-2008 ως και 17-8-2014 (δεν θα απαντήσω σε πμ)
Άβαταρ μέλους
AlexandrosG
Δημοσιεύσεις: 466
Εγγραφή: Πέμ Οκτ 22, 2009 5:31 am
Επικοινωνία:

Re: Όρια με ολοκληρώματα

#6

Μη αναγνωσμένη δημοσίευση από AlexandrosG » Τρί Μάιος 04, 2010 2:58 pm

Για το 1):

Από το διώνυμο του Νεύτωνα παίρνουμε δυο συμπεράσματα: 1+nx^2 <(1+x^2)^n και ότι ο παρονομαστής είναι πολυώνυμο του n βαθμού σίγουρα μεγαλύτερου του 1. Το πρώτο μαζί με το Θεώρημα Κυριαρχημένης Σύγκλισης του Lebesgue μας λέει ότι μπορούμε να εναλλάξουμε όριο και ολοκλήρωμα και το δεύτερο ότι η οριακή συνάρτηση είναι η μηδενική. Άρα το ζητούμενο όριο είναι ολοκλήρωμα της μηδενικής δηλαδή 0.

Νομίζω ότι η εναλλάγη γίνεται και με κάποιο ασθενέστερο κριτήριο που απαιτεί μόνο η συνάρτηση μέσα στο ολοκλήρωμα να είναι θετική.


Άβαταρ μέλους
silouan
Επιμελητής
Δημοσιεύσεις: 1398
Εγγραφή: Τρί Ιαν 27, 2009 10:52 pm

Re: Όρια με ολοκληρώματα

#7

Μη αναγνωσμένη δημοσίευση από silouan » Τρί Μάιος 04, 2010 5:31 pm

Κοτρώνης Αναστάσιος έγραψε: \displaystyle{6)} Αν η f:[0,1]\to\mathbb{R} είναι συνεχής και a,b>0, να βρεθεί το όριο \displaystyle{\lim_{t\to0^{+}}\int_{at}^{bt}\frac{f(x)}{x}\,dx}. (βγαίνει και λυκειακά)
Μια όχι λυκειακή λύση. Προσεγγίζουμε την f με ακολουθία πολυώνυμων από το θεώρημα Weirstrass. Οπότε αρκεί να βρούμε το όριο για πολυώνυμο. Σε αυτή την περίπτωση τα πράγματα είναι είναι εύκολα αφού όλα εκτός του τελευταίου όρου γίνονται 0, και ο τελευταίος δίνει ολοκλήρωμα a_0\ln\frac{b}{a}=p(0)\ln\frac{b}{a}


Σιλουανός Μπραζιτίκος
Άβαταρ μέλους
chris_gatos
Επιμελητής
Δημοσιεύσεις: 6962
Εγγραφή: Κυρ Δεκ 21, 2008 9:03 pm
Τοποθεσία: Ανθούπολη

Re: Όρια με ολοκληρώματα

#8

Μη αναγνωσμένη δημοσίευση από chris_gatos » Τρί Μάιος 04, 2010 5:32 pm

Για το 3)

Έχω:

\displaystyle{ 
\ln (x + \frac{{x^5 }}{t}) = \ln x + \ln (1 + \frac{{x^4 }}{t}),\forall x \in \left[ {1,2} \right],\forall t > 0 
}

Άρα:

\displaystyle{ 
\int\limits_1^2 {\ln (x + \frac{{x^5 }}{t})dx = \int\limits_1^2 {\ln xdx + \int\limits_1^2 {\ln (1 + \frac{{x^4 }}{t})dt = 2\ln 2 - 1 + \int\limits_1^2 {\ln (1 + \frac{{x^4 }}{t})dx} } } }  
}


Για σταθερό t>0 έχω πως η συνάρτηση

\displaystyle{ 
g(x)=1 + \frac{{x^4 }}{t},x \in \left[ {1,2} \right] 
}

είναι γνησίως αύξουσα στο [1,2]

Αρα:

\displaystyle{ 
g(1) \le g(x) \le g(2),\forall x \in \left[ {1,2} \right] \Leftrightarrow 1 + \frac{1}{t} \le 1 + \frac{{x^4 }}{t} \le 1 + \frac{{16}}{t},\forall x \in \left[ {1,2} \right] 
}

Τελικά:

\displaystyle{ 
\ln (1 + \frac{1}{t}) \le \ln (1 + \frac{{x^4 }}{t}) \le \ln (1 + \frac{{16}}{t}),\forall x \in \left[ {1,2} \right] 
}

Όλοκληρώνοντας ως προς χ:

\displaystyle{ 
\int\limits_1^2 {\ln (1 + \frac{1}{t})dx \le \int\limits_1^2 {\ln } (1 + \frac{{x^4 }}{t})dx}  \le \int\limits_1^2 {\ln (1 + \frac{{16}}{t})dx \Leftrightarrow } \ln (1 + \frac{1}{t})\int\limits_1^2 {dx}  \le \int\limits_1^2 {\ln (1 + \frac{{x^4 }}{t})dx}  \le \ln (1 + \frac{{16}}{t})\int\limits_1^2 {dx}  
}

Απ'όπου:

\displaystyle{ 
\ln (1 + \frac{1}{t}) \le \int\limits_1^2 {\ln (1 + \frac{{x^4 }}{t})dx}  \le \ln (1 + \frac{{16}}{t}) 
}

και

\displaystyle{ 
\begin{array}{l} 
 \mathop {\lim }\limits_{t \to \infty } \ln (1 + \frac{1}{t}) = 0 \\  
 \mathop {\lim }\limits_{t \to \infty } \ln (1 + \frac{{16}}{t}) = 0 \\  
 \end{array} 
}

Αρα και

\displaystyle{ 
\mathop {\lim }\limits_{t \to  + \infty } \int\limits_1^2 {\ln (1 + \frac{{x^4 }}{t})dx}  = 0 
}

Τελικό αποτέλεσμα για το ολοκλήρωμα:

2ln2-1.


Χρήστος Κυριαζής
Άβαταρ μέλους
chris_gatos
Επιμελητής
Δημοσιεύσεις: 6962
Εγγραφή: Κυρ Δεκ 21, 2008 9:03 pm
Τοποθεσία: Ανθούπολη

Re: Όρια με ολοκληρώματα

#9

Μη αναγνωσμένη δημοσίευση από chris_gatos » Τρί Μάιος 04, 2010 5:49 pm

Διαγράφω προσωρινά την απάντηση, μιας και είχα λάθος στην εφαρμογή του κριτηρίου παρεμβολής, όπως μου επεσήμανε και ο Νίκος (MathRider).

Πάμε για να επανορθώσουμε, αφού εδώ στη Χάλκη ανα δύο ημέρες δεν έχουμε ρεύμα...

Θα πάρω την ανισότητα :

\displaystyle{ 
\frac{{2x}}{\pi } \le \sin x \le x,x \in \left[ {0,\frac{\pi }{2}} \right] 
}

Το δεξί μέλος είναι πασίγνωστο.

Για το αριστερό χρησιμοποιήστε τη μονοτονία της συνάρτησης που παραπέμπει ο MathRider, παρακάτω και βγαίνει ομαλότατα.

Συνεπώς:

\displaystyle{ 
 - \frac{{2tx}}{\pi } \ge  - t\sin x \ge  - tx \Leftrightarrow e^{ - tx}  \le e^{ - t\sin x}  \le e^{ - \frac{{2tx}}{\pi }}  
}
Ολοκληρώνοντας ως πρό χ λαμβάνω:

\displaystyle{ 
\int\limits_0^{\frac{\pi }{2}} {e^{ - tx} dx}  \le \int\limits_0^{\frac{\pi }{2}} {e^{ - t\sin x} dx}  \le \int\limits_0^{\frac{\pi }{2}} {e^{ - \frac{{2tx}}{\pi }} dx}  
}

Υπολογίζοντας

\displaystyle{ 
\left[ {\frac{{-e^{ - tx} }}{t}} \right]_0^{\frac{\pi }{2}}  \le \int\limits_0^{\frac{\pi }{2}} {e^{ - t\sin x} dx}  \le \left[ { - \frac{{e^{ - \frac{{2tx}}{\pi }} }}{{\frac{{2t}}{\pi }}}} \right]_0^{\frac{\pi }{2}}  
}

Τελικά:

\displaystyle{ 
\frac{-1}{{te^{\frac{{\pi t}}{2}} }} + \frac{1}{t} \le \int\limits_0^{\frac{\pi }{2}} {e^{ - t\sin x} dx}  \le  - \frac{{\pi e^{ - \frac{{2tx}}{\pi }} }}{{2t}} + \frac{\pi }{{2t}} 
}

Λαμβάνοντας όρια για t->+oo εύκολα βλέπουμε πως τα άκρα τείνουν στο μηδέν.
Απο το κριτήριο παρεμβολής προκύπτει:

\displaystyle{ 
\mathop {\lim }\limits_{t \to  + \infty } \int\limits_0^{\frac{\pi }{2}} {e^{ - t\sin x} dx}  = 0 
}

Υ.Γ Συγνώμη για την ταλαιπωρία.
τελευταία επεξεργασία από chris_gatos σε Τετ Μάιος 05, 2010 3:34 pm, έχει επεξεργασθεί 3 φορές συνολικά.


Χρήστος Κυριαζής
Άβαταρ μέλους
chris_gatos
Επιμελητής
Δημοσιεύσεις: 6962
Εγγραφή: Κυρ Δεκ 21, 2008 9:03 pm
Τοποθεσία: Ανθούπολη

Re: Όρια με ολοκληρώματα

#10

Μη αναγνωσμένη δημοσίευση από chris_gatos » Τρί Μάιος 04, 2010 6:55 pm

Μια προσπάθεια επίλυσης για τη 2), με επιφύλαξη..

Ας θεωρήσω την ακολουθία συναρτήσεων fn με:

\displaystyle{ 
f_n (x) = \left\{ {\begin{array}{*{20}c} 
   {1,x = 0}  \\ 
   {\frac{{n\sin (\frac{x}{n})}}{{x(1 + x^2 )}},x > 0}  \\ 
\end{array}} \right. 
}


Η fn είναι συνεχής για κάθε χ μεγαλύτερο ή ίσο του μηδενός.


Παρατηρώ πως για χ διαφορετικό απο το μηδέν:

\displaystyle{ 
\mathop {\lim }\limits_{n \to \infty } |\frac{{\sin (\frac{x}{n})}}{{\frac{x}{n}(1 + x^2 )}} - \frac{1}{{1 + x^2 }}| = 0 
}

ενώ για χ=0 πάλι |fn-f|=0

Συνεπώς η ακολουθία συναρτήσεων fn με

\displaystyle{ 
f_n (x) = \frac{{n\sin (\frac{n}{x})}}{{x(1 + x^2 )}} 
}

συγκλινει ομοιόμορφα σε ένα διάστημα \displaystyle{ 
[0,t],t <  + \infty  
}

Συνεπώς (αφού όλες οι συναρτήσεις είναι συνεχείς στα αναφερθέντα διαστήματα), έχω:

\displaystyle{ 
\int\limits_0^t {\frac{{n\sin \left( {\frac{n}{x}} \right)}}{{x(1 + x^2 )}}dx\mathop  \to \limits^{o\mu } } \int\limits_0^t {\frac{1}{{1 + x^2 }}dx = \arctan t}  
}

Για t ->+οο έχω πως το ζητούμενο όριο είναι:

π/2.


Χρήστος Κυριαζής
Άβαταρ μέλους
chris_gatos
Επιμελητής
Δημοσιεύσεις: 6962
Εγγραφή: Κυρ Δεκ 21, 2008 9:03 pm
Τοποθεσία: Ανθούπολη

Re: Όρια με ολοκληρώματα

#11

Μη αναγνωσμένη δημοσίευση από chris_gatos » Τρί Μάιος 04, 2010 7:03 pm

Παραθέτω την προσπάθεια επίλυσης που έκανα για την 1). Δε μπορώ να πάω παρακάτω... Κάτι με τύπους Wallis μου θυμίζει. Τεσ'πα! ίσως κάποιος να εμπνευστεί απο εδώ και κάτω.

Έχω (με κατάλληλο χειρισμό):

\displaystyle{ 
\int\limits_0^1 {\frac{{1 + nx^2 }}{{(1 + x^2 )^n }}dx}  = \int\limits_0^1 {\frac{{1 + n(1 + x^2  - 1)}}{{(1 + x^2 )^n }}} dx = (1 - n)\int\limits_0^1 {\frac{1}{{(1 + x^2 )^n }}dx + n} \int\limits_0^1 {\frac{1}{{(1 + x^2 )^{n - 1} }}dx}  
}

Τώρα με κατάλληλη περιποίηση
( x=tant, \displaystyle{ 
dx = (1 + \tan ^2 t)dt 
}
Για χ=0 => t=0
Για x=1 => t=π/4)

και στα δύο ολοκληρώματα, προκύπτει:

\displaystyle{ 
(1 - n)\int\limits_0^{\frac{\pi }{4}} {(\cos t)^{2n - 2} dt}  + n\int\limits_0^{\frac{\pi }{4}} {(\cos t)^{2n - 4} dt}  
}


Χρήστος Κυριαζής
Math Rider
Δημοσιεύσεις: 137
Εγγραφή: Παρ Απρ 09, 2010 12:40 pm
Τοποθεσία: Πάτρα

Re: Όρια με ολοκληρώματα

#12

Μη αναγνωσμένη δημοσίευση από Math Rider » Τρί Μάιος 04, 2010 7:19 pm

Μια ιδέα για το υπολογισμό του 4) εδώ: viewtopic.php?f=54&t=6231


Νίκος Κ.
Άβαταρ μέλους
mathxl
Δημοσιεύσεις: 6736
Εγγραφή: Τρί Δεκ 23, 2008 3:49 pm
Τοποθεσία: Σιδηρόκαστρο
Επικοινωνία:

Re: Όρια με ολοκληρώματα

#13

Μη αναγνωσμένη δημοσίευση από mathxl » Τρί Μάιος 04, 2010 11:49 pm

Mια ιδέα για το 6 η οποία δεν ξέρω αν είναι σωστή.
Αν αλλάξουμε μεταβλητή x=ut τότε με εναλλαγή ορίου και ολοκληρώματος (θεώρημα κυριαρχημένης σύγκλισης) προκύπτει το ζητούμενο. Δεν ξέρω αν είναι σωστή γιατί απλά δεν ξέρω τις προυποθέσεις του θεωρήματος :mrgreen: .
Σχολική λύση δεν μπορώ να βρω !!


Ποτε δεν κάνω λάθος! Μια φορά νομιζα πως είχα κάνει, αλλά τελικά έκανα λάθος!
Απ' τα τσακάλια δεν γλυτώνεις μ' ευχές η παρακάλια. Κ. Βάρναλης
Aπέναντι στις αξίες σου να είσαι ανυποχώρητος

Ενεργό μέλος από 23-12-2008 ως και 17-8-2014 (δεν θα απαντήσω σε πμ)
Άβαταρ μέλους
Κοτρώνης Αναστάσιος
Επιμελητής
Δημοσιεύσεις: 3203
Εγγραφή: Κυρ Φεβ 22, 2009 11:11 pm
Τοποθεσία: Μπροστά στο πισί...
Επικοινωνία:

Re: Όρια με ολοκληρώματα

#14

Μη αναγνωσμένη δημοσίευση από Κοτρώνης Αναστάσιος » Τετ Μάιος 05, 2010 1:05 am

mathxl έγραψε:Mια ιδέα για το 6 η οποία δεν ξέρω αν είναι σωστή.
Αν αλλάξουμε μεταβλητή x=ut τότε με εναλλαγή ορίου και ολοκληρώματος (θεώρημα κυριαρχημένης σύγκλισης) προκύπτει το ζητούμενο. Δεν ξέρω αν είναι σωστή γιατί απλά δεν ξέρω τις προυποθέσεις του θεωρήματος :mrgreen: .
Σχολική λύση δεν μπορώ να βρω !!
Βασίλη η μια λύση που έχω κάνει είναι αυτή που λες. Μια σχολική προσέγγιση είναι η εξής :

\displaystyle{\int_{at}^{bt}\frac{f(x)}{x}\,dx\stackrel{\ln x=y}{=}\int_{\ln at}^{\ln bt}f(e^{y})\,dy=\ln\frac{b}{a}f(e^{\xi_{t}})} για κάποιο \displaystyle{\xi_{t}} με \displaystyle{\ln at<\xi_{t}<\ln bt}.

Όμως \displaystyle{t\to0^{+}\Rightarrow\xi_{t}\to-\infty\Rightarrow \ln\frac{b}{a}f(e^{\xi_{t}})\to\ln\frac{b}{a}f(0)} λόγω συνέχειας της \displaystyle{f}.


Εσύ....; Θα γίνεις κανίβαλος....;
Άβαταρ μέλους
Κοτρώνης Αναστάσιος
Επιμελητής
Δημοσιεύσεις: 3203
Εγγραφή: Κυρ Φεβ 22, 2009 11:11 pm
Τοποθεσία: Μπροστά στο πισί...
Επικοινωνία:

Re: Όρια με ολοκληρώματα

#15

Μη αναγνωσμένη δημοσίευση από Κοτρώνης Αναστάσιος » Τετ Μάιος 05, 2010 1:24 am

Μία υπόδειξη για το 8)
Δείξτε ότι αν \displaystyle{f} συνεχής και \displaystyle{g(x)} συνεχώς παραγωγίσιμη στο [a,b], τότε

\displaystyle{\int_{a}^{b}f(x)g{'}(x)=f(b)(g(b)-g(c_{x}))} για κάποιο \displaystyle{c_{x}\in(a,b)}.


Εσύ....; Θα γίνεις κανίβαλος....;
Άβαταρ μέλους
AlexandrosG
Δημοσιεύσεις: 466
Εγγραφή: Πέμ Οκτ 22, 2009 5:31 am
Επικοινωνία:

Re: Όρια με ολοκληρώματα

#16

Μη αναγνωσμένη δημοσίευση από AlexandrosG » Πέμ Μάιος 06, 2010 6:17 pm

Για την 8), άλλος τρόπος από αυτόν που προτείνει ο Αναστάσης:

Το παρακάτω σε πιο γενική μορφή αναφέρεται ως Λήμμα Riemann-Lebesgue:


Έστω f:[a,b] \to \mathbb{R} συνεχής. Τότε \displaystyle{\lim_{\nu \to + \infty} \int_a^bf(x)\sin(\nu x)dx=0}.


Στην άσκηση 8) η f είναι η \displaystyle{\frac{1}{x}}.


Άβαταρ μέλους
chris_gatos
Επιμελητής
Δημοσιεύσεις: 6962
Εγγραφή: Κυρ Δεκ 21, 2008 9:03 pm
Τοποθεσία: Ανθούπολη

Re: Όρια με ολοκληρώματα

#17

Μη αναγνωσμένη δημοσίευση από chris_gatos » Πέμ Μάιος 06, 2010 7:04 pm

Για την 8 κι εγώ.

Λήμμα:

Αν η συνάρτηση f είναι συνεχής και η συνάρτηση g μονότονη και έχει δεύτερη παράγωγο συνεχή στο [α,b] τότε ισχύει:

\displaystyle{ 
\int\limits_a^b {f(x)g(x)dx = g(a)\int\limits_a^\xi  {f(x)dx + g(b)\int\limits_\xi ^b {f(x)dx} } }  
}

για κάποιο ξ στο [α,b]


Λύση της άσκησης του Αναστάση:

Για f(x) θέτω sin(nx) και για g(x)=1/x. Ικανοποιούνται οι προυποθέσεις του λήμματος, άρα:

\displaystyle{ 
\int\limits_a^b {\sin (nx)\frac{1}{x}dx = \frac{1}{a}\int\limits_a^\xi  {\sin (nx)dx + \frac{1}{b}\int\limits_\xi ^b {\sin (nx)dx}  = \frac{1}{{na}}\left( {\cos (na} \right) - \cos (n\xi ))} }  - \frac{1}{{nb}}(\cos (n\xi ) - \cos (nb)) 
}

Λαμβάνοντας απόλυτα έχω:

\displaystyle{ 
|\int\limits_a^b {\sin (nx)\frac{1}{x}dx| = |\frac{1}{{na}}(\cos (na) - \cos (n\xi ))}  - \frac{1}{{nb}}(\cos (n\xi ) - \cos (nb))| \le \frac{2}{{na}} + \frac{2}{{nb}} < \frac{4}{{na}} 
}


Αφήνοντας το n ν τείνει στο άπειρο εύκολα προκύπτει πως το ζητούμενο όριο είναι μηδέν.
Η απόδειξη περιέχεται στα ''ολοκληρώματα'' του Κ.Kuratowski, τη θεώρησα ενδιαφέρουσα και σας την έδωσα. Συγνώμη για την απουσία απόδειξης του λήμματος μα νομίζω πως είναι αρκετά εύκολη εφαρμογή βασικών θεωρημάτων της ανάλυσης.


Χρήστος Κυριαζής
Άβαταρ μέλους
Demetres
Γενικός Συντονιστής
Δημοσιεύσεις: 8989
Εγγραφή: Δευ Ιαν 19, 2009 5:16 pm
Τοποθεσία: Λεμεσός/Πύλα
Επικοινωνία:

Re: Όρια με ολοκληρώματα

#18

Μη αναγνωσμένη δημοσίευση από Demetres » Πέμ Μάιος 06, 2010 11:12 pm

Να προσθέσω ότι αρκετά από αυτά μπορούν να αποδειχθούν ανταλλάζοντας την σειρά ορίου και ολοκληρώματος. Π.χ. για το (1) έχουμε

\displaystyle{\lim_{n \to \infty} \int_0^1 \frac{1+nx^2}{(1+x^2)^n} \; dx = \int_0^1 \lim_{n \to \infty}  \frac{1+nx^2}{(1+x^2)^n} \; dx = \int_0^1 g(x) \; dx = 0} όπου g(0) = 1 και g(x) = 0 για x \in (0,1].

Το όριο \displaystyle{ \lim_{n \to \infty}  \frac{1+nx^2}{(1+x^2)^n} } μπορεί να υπολογιστεί π.χ. με L' Hopital. (Προσοχή: Το θεωρούμε συνάρτηση του n και όχι του x.)

Μένει να δικαιολογηθεί η εναλλαγή ορίου και ολοκληρώματος. Σε αυτήν την περίπτωση δικαιολογείται από το θεώρημα κυριαρχημένης σύγκλισης επειδή για κάθε n και κάθε x \in [0,1] έχουμε \displaystyle{ \left| \frac{1 + nx^2}{(1+x^2)^n} \right| \leqslant 1} το οποίο είναι ανεξάρτητο του n.


lemonidas
Δημοσιεύσεις: 32
Εγγραφή: Κυρ Ιαν 10, 2010 11:09 am

Re: Όρια με ολοκληρώματα

#19

Μη αναγνωσμένη δημοσίευση από lemonidas » Παρ Μάιος 07, 2010 1:21 am

Για το δεύτερο μέλος της 9 (με κίνδυνο να φάω ξύλο από τον Αλέξανδρο για αυτό που θα χρησιμοποιήσω (πάλι :roll: )), μπορούμε να πάμε με Weierstrass.

Έστω L_n(f) = \displaystyle{\lim_{n\to+\infty}\frac{\displaystyle\int_{0}^{1}x^{n}f(x)\,dx}{\displaystyle\int_{0}^{1}x^{n}\,dx}}.

Αρκεί να υπολογίσουμε το \int_{0}^{1}x^{n+k}f(x)dx=\frac {1} {n+k+1}

Τώρα λόγω γραμμικότητας \displaystyle{\lim_{n\to+\infty} L_n(P)=P(1) για κάθε πολυώνυμο P.
Έστω ε>0.
Από θεώρημα Weierstrass μπορούμε να πάρουμε πολυώνυμο P ώστε |f(x)-p(x)|<\epsilon, \forall \epsilon \in [0,1]

Όμως λόγω του ανωτέρω ορίου μπορούμε να πάρουμε n_0 ώστε \forall n \geq n_0, |L_n(P)-P(1)|< \epsilon

Τέλος |L_n(f)-L_n(P)|\leq L_n(|f-p|) < L_n(\epsilon \cdot I) = \epsilon (Ι η I(x)=1)

Τώρα με μια τριγωνική ανισότητα για αρκετά μεγάλα n:

|L_n(f)-f(1)|\leq |L_n(f)-L_n(P)|+|L_n(P)-P(1)|+|f(1)-P(1)| \leq 3\epsilon


Engineers will go without food and hygiene for days to solve a problem. (Other times just because they forgot.) And when they succeed in solving the problem they will experience an ego rush that is better than sex- and I'm including the kind of sex where other people are involved.
Άβαταρ μέλους
silouan
Επιμελητής
Δημοσιεύσεις: 1398
Εγγραφή: Τρί Ιαν 27, 2009 10:52 pm

Re: Όρια με ολοκληρώματα

#20

Μη αναγνωσμένη δημοσίευση από silouan » Παρ Μάιος 07, 2010 1:41 am

Πολύ παρόμοιο με το θέμα που μας είχαν βάλει στο SEEMOUS το 2009. Ε Λεωνίδα ? Λύνεται νομίζω και με κρητήριο κυριαρχημένης σύγκλισης.


Σιλουανός Μπραζιτίκος
Απάντηση

Επιστροφή σε “ΑΝΑΛΥΣΗ”

Μέλη σε σύνδεση

Μέλη σε αυτήν τη Δ. Συζήτηση: Δεν υπάρχουν εγγεγραμμένα μέλη και 3 επισκέπτες